Difference between revisions of "2006 AMC 8 Problems/Problem 4"
Math Kirby (talk | contribs) (Created page with "== Problem == Initially, a spinner points west. Chenille moves it clockwise <math> 2 \dfrac{1}{4}</math> revolutions and then counterclockwise <math> 3 \dfrac{3}{4}</math> revolu...") |
AlcumusGuy (talk | contribs) |
||
Line 22: | Line 22: | ||
== Solution == | == Solution == | ||
If the spinner goes clockwise <math> 2 \dfrac{1}{4}</math> revolutions and then counterclockwise <math> 3 \dfrac{3}{4}</math> revolutions, it ultimately goes counterclockwise <math> 1 \dfrac{1}{2} </math> which brings the spinner pointing <math> \boxed{\textbf{(B)}\ \text{east}} </math>. | If the spinner goes clockwise <math> 2 \dfrac{1}{4}</math> revolutions and then counterclockwise <math> 3 \dfrac{3}{4}</math> revolutions, it ultimately goes counterclockwise <math> 1 \dfrac{1}{2} </math> which brings the spinner pointing <math> \boxed{\textbf{(B)}\ \text{east}} </math>. | ||
+ | |||
+ | {{AMC8 box|year=2006|num-b=3|num-a=5}} |
Revision as of 15:59, 21 November 2011
Problem
Initially, a spinner points west. Chenille moves it clockwise revolutions and then counterclockwise revolutions. In what direction does the spinner point after the two moves?
Solution
If the spinner goes clockwise revolutions and then counterclockwise revolutions, it ultimately goes counterclockwise which brings the spinner pointing .
2006 AMC 8 (Problems • Answer Key • Resources) | ||
Preceded by Problem 3 |
Followed by Problem 5 | |
1 • 2 • 3 • 4 • 5 • 6 • 7 • 8 • 9 • 10 • 11 • 12 • 13 • 14 • 15 • 16 • 17 • 18 • 19 • 20 • 21 • 22 • 23 • 24 • 25 | ||
All AJHSME/AMC 8 Problems and Solutions |